Logic Games Random Mix - - Question 20
If O is faster than F, then which one of the following chips could be ranked second?
Replies
Skylar August 4, 2020
@michellesheinker@gmail.com, happy to help!There is a video explanation posted for both this game setup and for this particular question. To access these, simply tap on the play symbol next to the word "Explanation" in the top right corner. I just checked and the video explanations work for me, but if for some reason they do not work for you, you may be experiencing technological issues. Please contact our support staff by tapping "support" from the left menu or by calling 855.483.7862 ext. 2 Monday-Friday 9am-6pm PT if this is the case.
In the meantime, I can provide you with a written walkthrough:
The question tells us that O > F. We combine this with what we know from Rules #4, #5, and #6 to conclude that H _ J and K _ _ L both come before O, which in turn comes before F.
Rule #2 tells us that either F or G is first. Since we now have variables that must precede F, this means that F cannot be first. Therefore, G must be first. This eliminates answer choice (A). G must be first, so it cannot be ranked second.
Let's look at answer choice (B). Is it possible to have H second?
G H __ __ __ __ __ __
1 2 3 4 5 6 7 8
We know from Rule #4 that if H is second, J must be fourth.
G H __ J __ __ __ __
1 2 3 4 5 6 7 8
We can also deduce that F must be eighth, since Rule #3 says M cannot be last and we are told that all of our other unplaced variables have at least one other variable they must precede.
G H __ J __ __ __ F
1 2 3 4 5 6 7 8
Rule #5 tells us that we must place a K __ __ L block, so we place K third and L sixth.
G H K J __ L __ F
1 2 3 4 5 6 7 8
Rule #6 tells us that J and L must both come before O, so O must go seventh. This leaves M to go fifth. Our final diagram looks like:
G H K J M L O F
1 2 3 4 5 6 7 8
This is a valid scenario in which H is second, so (B) is correct. We can stop here in the interest of time.
For the sake of explanation, I will discuss the remaining incorrect answers:
(C) is incorrect because it leaves no room for our H _ J and K _ _ L blocks to both fit before O. If we start out with our deductions that G must be first and F must be last in addition to the answer choice telling us that M must be second, we have:
G M __ __ __ __ __ F
1 2 3 4 5 6 7 8
We have 5 variables left to place with 5 spots open. We know from Rule #6 that O must come after J and L, so we can place O seventh. We now have 4 adjacent spots open to place H _ J and K _ _ L, which is impossible. Therefore, (C) is incorrect.
(D) is incorrect because if J is second, there is no way to arrange our H _ J block.
(E) is incorrect because if L is second, there is no way to arrange our K __ __ L block.
Does that make sense? Hope it helps! Please let us know if you have any other questions.
nashlurie February 26, 2021
There is still no video explanation / video explanations are out of order.JackM November 3, 2022
I don't see them eitherJackM November 3, 2022
I don't see them either.